Dérivabilité

Exercices du dossier Dérivabilité

Exercice 1329

9 novembre 2022 12:19 — Par Patrice Lassère

Bibliographie



[ID: 2531] [Date de publication: 9 novembre 2022 12:19] [Catégorie(s): Géométrie Topologie Continuité Dérivabilité Intégration Suites et séries Suites et séries de fonctions, séries entières Fonctions holomorphes Calcul différentiel Equations différentielles Analyse fonctionnelle Combinatoires et probabilités En cours... ] [ Nombre commentaires: 0] [nombre d'éditeurs: 1 ] [Editeur(s): Emmanuel Vieillard-Baron ] [nombre d'auteurs: 1 ] [Auteur(s): Patrice Lassère ]
Accordéon
Titre
Solution
Texte

Existence d’un opérateur à la dérivée de Dirac sur \(\mathscr C^0(\mathbb R,\mathbb R)\) *

9 novembre 2022 15:18 — Par Patrice Lassère

Déterminer les formes linéaires \(D\) sur \(\mathscr C^0(\mathbb R,\mathbb R)\) vérifiant \[D(fg)=f(0)D(g)+D(f)g(0),\quad\forall\,f,g\in\mathscr C^0(\mathbb R,\mathbb R).\]



[ID: 2679] [Date de publication: 9 novembre 2022 15:18] [Catégorie(s): Dérivabilité ] [ Nombre commentaires: 0] [nombre d'éditeurs: 1 ] [Editeur(s): Emmanuel Vieillard-Baron ] [nombre d'auteurs: 1 ] [Auteur(s): Patrice Lassère ]
Accordéon
Titre
Solution
Texte

Existence d’un opérateur à la dérivée de Dirac sur \(\mathscr C^0(\mathbb R,\mathbb R)\)
Par Patrice Lassère le 9 novembre 2022 15:18

\(\mathbf{1}\) désignant la fonction constante \(x\mapsto\mathbf{1}(x)=1\) nous avons \(D(\mathbf{1}.\mathbf{1})=D(\mathbf{1})=D(\mathbf{1})+D(\mathbf{1})\), soit \(D(\mathbf{1})=0\) et par suite \(D\) est nulle sur toutes les applications constantes. Comme pour toute \(f\in\mathscr C^0(\mathbb R,\mathbb R)\), \(f=(f-f(0))+f(0)\ \Rightarrow \ D(f)=D(f-f(0))\) il est suffisant de se concentrer sur la restriction de \(D\) sur les applications nulles à l’origine ; pour une telle application \(D(f^2)=0\) si bien que pour \(f\geq 0\) : \(D(f)=D((\sqrt{f})^2)=0\). Dans le cas général, comme il est toujours possible d’écrire \(f\) comme différence de deux fonctions continues, positives et nulles à l’origine (\(f=\max\{f(x),0\}-\max\{-f(x),0\}\)) on a encore \(D(f)=0\) et la seule forme linéaire qui convienne est la forme identiquement nulle.


Deux fonctions \(f,g\) dérivables telles que \(f'g'\) ne soit pas une dérivéee *

9 novembre 2022 15:18 — Par Patrice Lassère

Montrer qu’il existe deux fonction dérivables \(f,g\ :\ \mathbb R\to\mathbb R\) telles que \(f'g'\) ne soit pas une dérivée.



[ID: 2681] [Date de publication: 9 novembre 2022 15:18] [Catégorie(s): Dérivabilité ] [ Nombre commentaires: 0] [nombre d'éditeurs: 1 ] [Editeur(s): Emmanuel Vieillard-Baron ] [nombre d'auteurs: 1 ] [Auteur(s): Patrice Lassère ]
Accordéon
Titre
Solution
Texte

Deux fonctions \(f,g\) dérivables telles que \(f'g'\) ne soit pas une dérivéee
Par Patrice Lassère le 9 novembre 2022 15:18

Considérons1 les applications \(f,g\ :\ \mathbb R\to\mathbb R\) définie par \[f(x)=\begin{cases} x^2\sin(1/x) \quad &\text{ si }\quad x\neq 0\\ 0 \quad &\text{ si }\quad x=0,\end{cases}\] et \[g(x)=\begin{cases} 2x\sin(1/x)\quad &\text{ si }\quad x\neq 0\\ 0 \quad &\text{ si }\quad x=0.\end{cases}\] \(f\) est dérivable sur \(\mathbb R\) avec \[f'(x)=\begin{cases} 2x\sin(1/x)-\cos(1/x) \quad &\text{ si }\quad x\neq 0\\ 0 \quad &\text{ si }\quad x=0,\end{cases}\] \(g\) est continue sur \(\mathbb R\), c’est donc une dérivée. Par conséquent, \(h:=g-f'\) est une dérivée : il existe donc une application dérivable \(H\ :\ \mathbb R\to\mathbb R\) telle que \[H'(x)=h(x)=g(x)-f'(x)=\begin{cases} \cos(1/x) \quad &\text{ si }\quad x\neq 0\\ 0 \quad &\text{ si }\quad x=0.\end{cases}\] Considérons alors \[h^2(x)=\begin{cases} \dfrac{1+\cos(2/x)}{2} \quad &\text{ si }\quad x\neq 0\\ 0 \quad &\text{ si }\quad x=0,\end{cases}\] nous allons vérifier que \(h^2\) n’est pas une dérivée et répond donc à notre problème. Supposons qu’il existe une application dérivable \(D\ :\ \mathbb R\to\mathbb R\) telle que \(D'=h^2\), en notant \(\tilde H(x)=H(x/2)\) nous avons \[\tilde H'(x)=\begin{cases} \dfrac{\cos(2/x)}{2} \quad &\text{ si }\quad x\neq 0\\ 0 \quad &\text{ si }\quad x=0,\end{cases}\] qui implique \[D'(x)=\begin{cases} \dfrac{1}{2}+\tilde H'(x) \quad &\text{ si }\quad x\neq 0\\ \tilde H'(x) \quad &\text{ si }\quad x=0,\end{cases}\] ou encore \[D'(x)-\tilde H'(x) =\left( D(x)-\tilde H(x) \right)' = \begin{cases} \dfrac{1}{2} \quad &\text{ si }\quad x\neq 0\\ 0 \quad &\text{ si }\quad x=0.\end{cases}\] Mais il est bien connu (cf autre exo) qu’une dérivée ne peut avoir une discontinuité de première espèce : contradiction et l’application \(h^2\) est bien sans primitive.


  1. Wilkosz W. Fundamenta Mathematicae, (2)-1921.↩︎


Dérivation *

9 novembre 2022 15:18 — Par Patrice Lassère

Soit \(f\in\mathscr C^3(\mathbb R,\mathbb R)\). Montrer qu’il existe \(a\in\mathbb R\) tel que \[f(a)f'(a)f''(a)f'''(a)\geq 0.\]



[ID: 2683] [Date de publication: 9 novembre 2022 15:18] [Catégorie(s): Dérivabilité ] [ Nombre commentaires: 0] [nombre d'éditeurs: 1 ] [Editeur(s): Emmanuel Vieillard-Baron ] [nombre d'auteurs: 1 ] [Auteur(s): Patrice Lassère ]
Accordéon
Titre
Solution
Texte

Dérivation
Par Patrice Lassère le 9 novembre 2022 15:18

Si l’une des quatre applications \(f^{(i)}, 0\leq i\leq 3\) n’est pas de signe constant le problème est trivial. Supposons donc \(f,f',f'',f'''\) de signe constant, alors \(f\) et \(f''\) sont de même signe. Pour cela, si \(f''>0\) la formule de Taylor-Lagrange nous donne pour tout \(x\in\mathbb R\) \[f(x)=f(0)+xf'(0)+\dfrac{x^2}{2}f''(c_x)>f(0)+xf'(0)\] et \(f(0)+xf'(0)\) est certainement positif pour \(x\) suffisament grand et du signe de \(f'(0)\) : \(f\) étant supposée de signe constant \(f>0\) sur \(\mathbb R\)(on procède de manière analogue si \(f''<0\)). Ainsi \(f(x)f''(x)>0,\ \forall\,x\in\mathbb R\). Ce même raisonnement vaut pour le couple \(f', f'''\) et le résultat est démontré.

  Remarque : L’hypothèse \(\mathscr C^3\) est en fait superflue : trois fois dérivable suffit si l’on se souviens qu’une dérivée possède toujours la propriété des valeurs intermédiaires.


Approche matricielle du théorème des accroissements finis *

9 novembre 2022 15:18 — Par Patrice Lassère

Soient \(f,g,h\) trois fonctions continues sur \([a,b]\), dérivable sur \(]a,b[\). On définit

\[F(x)=\det\begin{pmatrix} f(x)&g(x)&h(x)\\ f(a)&g(a)&h(a)\\ f(b)&g(b)&h(b)\end{pmatrix}\]

montrer qu’il existe \(c\in]a,b[\) tel que \(F'(c)=0\), en déduire le théorème des accroissements finis puis la forme généralisée de ce théorème.



[ID: 2685] [Date de publication: 9 novembre 2022 15:18] [Catégorie(s): Dérivabilité ] [ Nombre commentaires: 0] [nombre d'éditeurs: 1 ] [Editeur(s): Emmanuel Vieillard-Baron ] [nombre d'auteurs: 1 ] [Auteur(s): Patrice Lassère ]
Accordéon
Titre
Solution
Texte

Approche matricielle du théorème des accroissements finis
Par Patrice Lassère le 9 novembre 2022 15:18

\(F\) est clairement continue sur \([a,b]\), dérivable sur \(]a,b[\) avec

\[F'(x)=\det\begin{pmatrix} f'(x)&g'(x)&h'(x)\\ f(a)&g(a)&h(a)\\ f(b)&g(b)&h(b)\end{pmatrix}\]

et vu les propriétés classiques du déterminant \(F(a)=F(b)=0\). Le théorème de Rolle assure alors l’existence de \(c\in]a,b[\) tel que \(F'(c)=0\).

Avec \(g(x)=x\) et \(h\equiv 1\) il vient

\[F'(c)=\det\begin{pmatrix} f'(c)&1&0\\ f(a)&a&1\\ f(b)&b&1\end{pmatrix}=0\]

soit \(f(b)-f(a)=f'(c)(b-a)\) i.e. le théorème des accroissements finis.

Si maintenant on choisit \(h\equiv 1\)

\[F'(c)=\det\begin{pmatrix} f'(c)&g'(c)&0\\ f(a)&g(a)&1\\ f(b)&g(b)&1\end{pmatrix}=0\]

nous donne

\[\exists\,c\in]a,b[\ :\ g'(c)(f(b)-f(a))=f'(c)(g(b)-g(a))\]

version forte du théorème des accroissements finis.


Comportement asymptotique du point intermédiaire dans la formule de Taylor-Lagrange *

9 novembre 2022 15:18 — Par Patrice Lassère

Soit \(f\,:\,\mathbb R\to\mathbb R\) une application \(n\) fois dérivable. Fixons \(x\in\mathbb R\), alors pour \(h>0\) la formule de Taylor-Lagrange assure de l’existence d’un réel \(\theta(h)\) tel que

\[f(x+h)=f(x)+hf'(x)+\dots+{{h^{n-1}}\over{(n-1)!}}f^{(n-1)}(x)+{{h^n}\over{n!}}f^{(n)}(x+\theta(h)h).\] Si de plus \(f^{(n+1)}(x)\) existe et est différent de zéro,montrer que

\[\lim_{h\to 0}\theta(h)={1\over n+1}.\]



[ID: 2687] [Date de publication: 9 novembre 2022 15:18] [Catégorie(s): Dérivabilité ] [ Nombre commentaires: 0] [nombre d'éditeurs: 1 ] [Editeur(s): Emmanuel Vieillard-Baron ] [nombre d'auteurs: 1 ] [Auteur(s): Patrice Lassère ]
Accordéon
Titre
Solution
Texte

Comportement asymptotique du point intermédiaire dans la formule de Taylor-Lagrange
Par Patrice Lassère le 9 novembre 2022 15:18

Par Taylor-Young

\[f(x+h)=f(x)+hf'(x)+\dots+{{h^{n}}\over{n!}}f^{(n)}(x)+ {{h^{n+1}}\over{(n+1)!}}f^{(n+1)}(x)+o(h^{n+1})\]

et par Taylor-Lagrange

\[f(x+h)=f(x)+hf'(x)+\dots+{{h^{n-1}}\over{(n-1)!}}f^{(n-1)}(x)+{{h^n}\over{n!}}f^{(n)}(x+\theta(h)h)\]

soit

\[{{f^{(n)}(x+\theta(h)h)-f^{(n)}(x)}\over h}={{f^{(n+1)}(x)}\over n+1}+{{o(h)}\over h}\]

et

\[\theta(h)={{{{f^{(n+1)}(x)}\over n+1}+{{o(h)}\over h}}\over{{{f^{(n)}(x+\theta(h)h)-f^{(n)}(x)}\over \theta(h)h}}}\]

puisque \(f^{(n+1)}(x)\ne 0\) on peut passer à la limite dans cette dernière égalité et le résultat suit.


Trois preuves du théorème de Darboux *

9 novembre 2022 15:18 — Par Patrice Lassère

Soit \(I\) un intervalle de \(\mathbb R\). Si \(f\,:\,I\to\mathbb R\) est dérivable sur \(I\) alors \(f'\) vérifie la propriété des valeurs intermédiaires.



[ID: 2689] [Date de publication: 9 novembre 2022 15:18] [Catégorie(s): Dérivabilité ] [ Nombre commentaires: 0] [nombre d'éditeurs: 1 ] [Editeur(s): Emmanuel Vieillard-Baron ] [nombre d'auteurs: 1 ] [Auteur(s): Patrice Lassère ]
Accordéon
Titre
Solution
Texte

Trois preuves du théorème de Darboux
Par Patrice Lassère le 9 novembre 2022 15:18

Soient \(x<y\) dans \(I\), et les applications \(\varphi,\psi\,:\,[x,y]\to\mathbb R\) définies par

\[\varphi(t)=\begin{cases}{{f(x)-f(t)}\over {x-t}}&\text{ si }t\in]x,y]\\ f'(x)&\text{ si } t=x,\\ \end{cases} \quad \psi(t)=\begin{cases}{{f(y)-f(t)}\over {y-t}}&\text{ si }t\in[x,y[\\ f'(y)&\text{ si } t=y,\\ \end{cases}\]

vu les hypothèses sur \(f\), \(\varphi\) et \(\psi\) sont continues sur \([x,y]\) et donc \(I_\varphi:=\varphi([x,y])\) et \(I_\psi:=\psi([x,y])\) sont deux intervalles de \(\mathbb R\). Ils sont d’intersection non vide puisque \(\varphi(y)=\psi(x)\), par conséquent \(I_\varphi\cup I_\psi\) est un intervalle non vide de \(\mathbb R\) (la réunion de deux connexes non disjoints est toujours connexe). Ainsi, pour tout réel \(\lambda\in\left(f'(x),f'(y)\right)\) il existe \(t\in]x,y[\) tel que \(\varphi(t)=\lambda\) ou \(\psi(t)=\lambda\), si par exemple \(\varphi(t)=\lambda\) (idem avec \(\psi\)) le théorème des accroissements finis nous assure qu’il existe \(\zeta_t\in]x,y[\) tel que

\[\lambda=\varphi(t)={{f(x)-f(t)}\over{x-t}}=f'(\zeta_t) .\]

finalement

\[\forall\,x,y \in I, \ \forall\, \lambda\in\left(f'(x),f'(y)\right)\quad\exists\,\zeta\in]x,y[\ :\ f'(\zeta)=\lambda\]

et \(f'\) possède bien la propriété des valeurs intermédiaires. CQFD

Supposons \(I\) sans borne supérieure (i.e. \(I=[a,b[\) ou \(]a,b[\)). Considérons la partie connexe \(\mathscr C=\{(x,y)\in I\times I\ :\ x\ne y\}\) l’application

\[\varphi\,:\ (x,y)\in\mathscr C\mapsto \varphi(x,y)={{f(x)-f(y)}\over{x-y}},\]

est visiblement continue, \(\varphi(\mathscr C)\) est donc connexe dans \(\mathbb R\) : c’est un intervalle, et par le théorème des accroissements finis \(\varphi(\mathscr C)\subset f'(I)\). Inversement, vu la forme de \(I\) on a, pour tout \(x\in I\)

\[f'(x)=\lim_{y\to x,\,y>x}\varphi(x,y)\ \in\ \overline{\varphi(\mathscr C)}\]

en resumé \(\varphi(\mathscr C)\) est connexe et

\[\varphi(\mathscr C)\subset f'(I)\subset \overline{\varphi(\mathscr C)}\]

qui implique (cours sur les connexes) la connexité de \(f'(I)\) qui assure notre résultat.

On procéde de même si \(I=]a,b]\) et si \(I=[a,b]\) on écrit \([a,b]=]a,b]\cup[a,b[\).

soient

\[a,b\in I,\ \lambda\in\mathbb R\quad\text{ tels que } \quad f'(a)<\lambda<f'(b)\]

et

\[\varphi\,:\,x\in I\mapsto \varphi(x)=f(x)-\lambda x\in\mathbb R.\]

\(\varphi\) est dérivable sur \(I\) et il existe \(\alpha,\beta\in]a,b[\) tels que \(\varphi(\alpha)=\varphi(\beta)\) (sinon \(\varphi\) est injective continue sur \(]a,b[\) est strictement monotone, \(\varphi\) est dérivable : \(\varphi'\) est de signe constant ce qui est absurde puisque \(\varphi'(a)=f'(a)-\lambda<0\) et \(\varphi'(b)=f'(b)-\lambda>0\)...) on conclut alors avec le théorème de Rolle.


Accordéon
Titre
Solution
Texte

Sur le point d’inflexion
Par Patrice Lassère le 9 novembre 2022 15:18

On montre facilement que \(f\) est deux fois dérivable en \(x=0\) avec \(f'(0)=f''(0)=0\). En outre \(x=0\) est bien point d’inflexion de \(f\) car \(f\) est \(>0\) sur \(\mathbb R_+^\star\) et \(<0\) sur \(\mathbb R_-^\star\) alors que la tangente à l’origine au graphe de \(f\) est l’axe des abcisses. Toutefois, après un petit calcul on a au voisinage de l’origine \(f''(x)=-x\sin(x^{-1})+o(x)\) qui n’est bien sûr pas de signe constant sur aucun voisinage à droite (et à gauche) de zéro.

Remarque : si \(f\) est deux fois dérivable au voisinage d’un point \(a\) et si sa dérivée seconde s’y annule en changeant de signe alors \(f\) admet un point d’inflexion en \(a\). La réciproque est donc fausse.


Rolle sur \(\mathbf{\mathbb R}\) *

9 novembre 2022 15:18 — Par Patrice Lassère

Soit \(f\,:\mathbb R\to\mathbb R\) une fonction dérivable telle que

\[\lim_{+\infty}f(x)=\lim_ {-\infty}f(x)=l\in\mathbb R.\]

Montrer qu’il existe \(c\in\mathbb R\) tel que \(f'(c)=0\).



[ID: 2693] [Date de publication: 9 novembre 2022 15:18] [Catégorie(s): Dérivabilité ] [ Nombre commentaires: 0] [nombre d'éditeurs: 1 ] [Editeur(s): Emmanuel Vieillard-Baron ] [nombre d'auteurs: 1 ] [Auteur(s): Patrice Lassère ]
Accordéon
Titre
Solution
Texte

Rolle sur \(\mathbf{\mathbb R}\)
Par Patrice Lassère le 9 novembre 2022 15:18

On se ramène au cas classique en posant si \(\displaystyle x\in\large]{-\pi\over 2},{\pi\over 2}\large[,\ g(x)=f(\tan(x))\) et \(\displaystyle g(x)=l\) pour \(\displaystyle x=\pm{\pi\over 2}\). On conclut en appliquant le théorème de Rolle à \(g\).


Dérivabilité et accroissements finis *

9 novembre 2022 15:18 — Par Patrice Lassère

Soit \(f\,:\ [a,b)\to\mathbb R\) une application dérivable. Montrer que \(f'(a)\) est valeur d’adhérence de \(f'(]a,b[)\).



[ID: 2695] [Date de publication: 9 novembre 2022 15:18] [Catégorie(s): Dérivabilité ] [ Nombre commentaires: 0] [nombre d'éditeurs: 1 ] [Editeur(s): Emmanuel Vieillard-Baron ] [nombre d'auteurs: 1 ] [Auteur(s): Patrice Lassère ]
Accordéon
Titre
Solution
Texte

Dérivabilité et accroissements finis
Par Patrice Lassère le 9 novembre 2022 15:19

Il faut donc montrer que \(\forall\,\varepsilon>0,\,\exists\,\eta>0\,:\ \exists \zeta\in]a,a+\eta[\text{ tel que }\vert f'(a)-f'(\zeta)\vert\leq \varepsilon\). \(f\) étant dérivable (à droite)

\[\qquad\forall\,\varepsilon>0,\,\exists\,\eta>0\,:\ \forall\,x\in ]a,a+\eta[\ :\ \left\vert {{f(x)-f(a)}\over{x-a}}-f'(a)\right\vert\leq\varepsilon, {(\text{\ding{56}})}\]

mais pour un tel \(x\) le théorème des accroissements finis assure de l’existence d’un \(\zeta\in]a,x[\) tel que \(f(x)-f(a)=(x-a)f'(\zeta)\). Il ne reste plus qu’à reporter dans () pour conclure.

Remarques : les discontinuités de l’application dérivée \(f'\) ne sont donc pas arbitraires, par exemple une discontinuité de première espèce (un saut) est proscrite pour une dérivée. Il faut d’ailleurs se souvenir du théorème de qui dit que \(f'\) possède la propriété des valeurs intermédiaires (on peut aussi traiter cet exercice avec Darboux).

Tant que nous y sommes, il est essentiel pour la leçon sur la dérivabilité d’avoir un exemple d’une fonction dérivable en un point à dérivée non continue, l’exemple canonique étant

\[f(x)= \begin{cases} x^2\sin(x^{-1}) \quad \ \,\text{ si }x\ne 0,\\ 0\qquad\qquad\qquad\text{ si } x=0.\end{cases}\]


Dérivabilité et accroissements finis *

9 novembre 2022 15:19 — Par Patrice Lassère

Soit \(f\in\mathscr C^0([a,b],\mathbb R)\) une application dérivable sur \(]a,b[\) sauf peut être en un point \(c\in]a,b[\). Si \(f'(x)\) admet une limite \(l\) lorsque \(x\) tend vers \(c\), montrer que \(f\) est dérivable en \(c\) et \(f'(c)=l\).



[ID: 2697] [Date de publication: 9 novembre 2022 15:19] [Catégorie(s): Dérivabilité ] [ Nombre commentaires: 0] [nombre d'éditeurs: 1 ] [Editeur(s): Emmanuel Vieillard-Baron ] [nombre d'auteurs: 1 ] [Auteur(s): Patrice Lassère ]
Accordéon
Titre
Solution
Texte

Dérivabilité et accroissements finis
Par Patrice Lassère le 9 novembre 2022 15:19

Soit \(a<x<c\), appliquons le théorème des accroissements finis à \(f\) sur \([x,c]\) : il existe \(x<\eta_x<c\) tel que \[\quad {{f(x)-f(c)}\over{x- c}}=f'(\eta_x).\] Mais \[(x<\eta_x<c)\quad\Longrightarrow\quad \left( \lim_{x\to c_-}\eta_x=c\right)\] donc, vu les hypothèses sur \(f\) :

\[\lim_{x\to c_-} {{f(x)-f(c)}\over{x-c}} =\lim_{x\to c_-}f'(\eta_x)=\lim_{x\to c_-}f(x)=l.\]

\(f\) est donc dérivable à gauche au point \(c\) avec \(f_g'(c)=l\), on fait de même à droite.


Toute application convexe et majorée sur \(\mathbb R\) est constante *

9 novembre 2022 15:19 — Par Patrice Lassère

Soit \(f\ :\,\mathbb R\to\mathbb R\) deux fois dérivable, convexe et majorée : montrer que \(f\) est constante.



[ID: 2699] [Date de publication: 9 novembre 2022 15:19] [Catégorie(s): Dérivabilité ] [ Nombre commentaires: 0] [nombre d'éditeurs: 1 ] [Editeur(s): Emmanuel Vieillard-Baron ] [nombre d'auteurs: 1 ] [Auteur(s): Patrice Lassère ]
Accordéon
Titre
Solution
Texte

Toute application convexe et majorée sur \(\mathbb R\) est constante
Par Patrice Lassère le 9 novembre 2022 15:19

Si \(f\) n’est pas constante, on peut trouver \(a\in\mathbb R\) tel que \(f'(a)\ne 0\) et la formule de Taylor-Lagrange nous donne pour \[x\in\mathbb R,\ \exists\,\zeta_x\in(a,x)\quad:\quad f(x+a)=f(a)+xf'(a)+{x^2\over 2}f''(\zeta_x)\geq f(a)+xf'(a)\] la dernière inégalité résultant du fait que \[\left( f\ \text{ deux fois dérivable et convexe} \right) \quad\Longrightarrow\quad\left( f''\geq 0\right) .\] Si par exemple \(f'(a)>0\) on obtient alors une contradiction en faisant tendre \(x\) vers \(+\infty\) (et vers \(-\infty\) si \(f'(a)<0\)...)

Remarques :  Si la fonction est seulement convexe le résultat bien entendu subsiste, il faut juste être un peu plus délicat : si \(f\) est non constante, soient \(a<b\) vérifiant \(f(a)<f(b)\) ou \(f(a)>f(b)\) et pour tout \(a<b<x\)

\[\begin{aligned} f(b)&=f\left({{x-b}\over{x-a}}a+{{b-a}\over{x-a}}x\right)\\ &\leq {{x-b}\over{x-a}}f(a)+{{b-a}\over{x-a}}f(x), \end{aligned}\]

soit

\[\forall\, x>b>a,\qquad f(x)\geq {{x-a}\over{b-a}}f(b)-{{x-b}\over{b-a}}f(a).\qquad{(\bigstar)}\]

Si \(f(b)>f(a)\) il existe \(\delta>0\) tel que \(f(b)=f(a)+\delta\) et \((\bigstar)\) devient pour tout \(x>b\) :

\[\begin{aligned} f(x)&\geq {{x-a}\over{b-a}}f(b)-\left({{x-a}\over{b-a}}+{{a-b}\over{b-a}}\right)f(a)\\ &={{x-a}\over{b-a}}\left(f(b)-f(a)\right)+f(a)\\ &\geq {{x-a}\over{b-a}}\delta+f(a):=h(x) \end{aligned}\]

la fonction \(h\) est non majorée sur \(]b,+\infty[\), il en est donc de même pour \(f\) d’où la contradiction. On procède de manière analogue si \(f(b)<f(a)\) en établissant pour \(x<a<b\) avec \(\delta:=f(a)-f(b)\)

\[f(x)\geq {{b-x}\over{b-a}}\delta+f(b)\]

Ce résultat ne subsiste plus si on remplace \(\mathbb R\) par un intervalle de la forme \((a,+\infty[\) (resp. \(]-\infty,a)\)), il suffit par exemple de considérer \(f(x) =e^{-x}\) (resp. \(f(x)=e^x\)).


Accordéon
Titre
Solution
Texte

Régularité et existence de développement limité en un point
Par Patrice Lassère le 9 novembre 2022 15:19

Pour la continuité et la dérivabilité c’est classique. En outre pour tout \(n\in\mathbb N\) \(f(x)=o(x^n)\) à l’origine : elle admet donc un développement limité à l’ordre \(n\) en ce point (la partie principale étant le polynôme nul...).

Remarques : Une fonction est continue en un point, si et seulement si elle admet un développement limité à l’ordre zéro en ce point ; elle y est dérivable si et seulement si elle y admet un développement limité à l’ordre \(1\). Si une fonction est \(n\) fois dérivable en un point alors (Taylor-Lagrange ou Young ) elle admet un développement limité d’ordre \(n\) en ce point et l’exemple précédent nous montre que pour \(n\geq 2\) la réciproque est fausse.

On a aussi pour \(n=2\) le contre-exemple canonique : \(\displaystyle f(x)=\begin{cases} x^3\sin(x^{-1})\quad&\text{ si }x\in\mathbb R^\star,\\ 0\quad&\text{ sinon.}\end{cases}\)


Parité, dérivabilité et développement limité *

9 novembre 2022 15:19 — Par Patrice Lassère

On définit \(f\) sur \(\mathbb R\) par

\[f(x)=\begin{cases} \exp(-{1\over x^2})+\cos(x) \quad&\text{ si }x> 0,\\ \cos(x)\quad&\text{ sinon}.\end{cases}\]

Montrer que \(f\) n’est pas paire mais que tous ses développements limités de \(f\) à l’origine sont sans termes de degré impair.



[ID: 2703] [Date de publication: 9 novembre 2022 15:19] [Catégorie(s): Dérivabilité ] [ Nombre commentaires: 0] [nombre d'éditeurs: 1 ] [Editeur(s): Emmanuel Vieillard-Baron ] [nombre d'auteurs: 1 ] [Auteur(s): Patrice Lassère ]
Accordéon
Titre
Solution
Texte

Parité, dérivabilité et développement limité
Par Patrice Lassère le 9 novembre 2022 15:19

Comme dans l’exercice précédent pour tout \(n\in\mathbb N\) : \[f(x)=\begin{cases} \cos(x)+o(x^n) \quad&\text{ si }x> 0,\\ \cos(x)\quad&\text{ sinon}.\end{cases}\] au voisinage de \(0_+\). Ainsi, \(f\) admet un développement limité à tout ordre à l’origine et c’est celui de la fonction cosinus : il est donc sans termes impairs.

Toufefois \(f\) n’est ni paire ni impaire puisque \[\vert f(x)-f(-x)\vert=\exp(-{1\over x^2})>0,\ \forall\,x\in\mathbb R^\star.\]

  Remarque : \(f\) n’est donc pas développable en série entière à l’origine.


Convexité et Accroisements Finis *

9 novembre 2022 15:19 — Par Patrice Lassère

Soit \(f\ :\ ]a,b[\to\mathbb R\) une application dérivable vérifiant : \[\forall\,x\neq y\ \text{dans}\ ]a,b[,\quad\exists\, !\ \zeta\in]a,b[\ \text{tel que}\quad \dfrac{f(x)-f(y)}{x-y}=f'(\zeta),\] montrer que \(f\) est soit strictement convexe, soit strictement concave sur \(]a,b[\).



[ID: 2705] [Date de publication: 9 novembre 2022 15:19] [Catégorie(s): Dérivabilité ] [ Nombre commentaires: 0] [nombre d'éditeurs: 1 ] [Editeur(s): Emmanuel Vieillard-Baron ] [nombre d'auteurs: 1 ] [Auteur(s): Patrice Lassère ]
Accordéon
Titre
Solution
Texte

Convexité et Accroisements Finis
Par Patrice Lassère le 9 novembre 2022 15:19

Supposons au contraire que \(f\) ne soit ni strictement convexe, ni strictement concave sur \(]a,b[\). Il existe alors deux réels \(a<x_1<x_2<b\) tels que le segment reliant les points \((x_1,f(x_1))\) et \((x_2,f(x_2))\) rencontre encore le graphe de \(f\) en au moins un point \((x_3,f(x_3))\) avec \(a<x_1<x_3<x_2<b\). Par hypothèse il existe deux uniques réels \(y_1,y_2\in]a,b[\) vérifiant \[\dfrac{f(x_3)-f(x_1)}{x_3-x_1}=f'(y_1)\quad\text{et}\quad \dfrac{f(x_2)-f(x_3)}{x_2-x_3}=f'(y_2),\] et le théorème des accroissement finis nous assure que \(x_1<y_1<x_3<y_2<x_2\), soit \(y_1\neq y_2\). D’un autre coté la colinéarité de \((x_1,f(x_1))\), \((x_2,f(x_2))\) et \((x_3,f(x_3))\) assure que \[f'(y_1)=\dfrac{f(x_3)-f(x_1)}{x_3-x_1}= \dfrac{f(x_2)-f(x_3)}{x_2-x_3}=f'(y_2),\] ce qui est contraire à l’hypothèse sur \(f\). D’où le résultat.


Zéros des dérivées d’une fonctions \(\mathscr C^\infty\) à support compact *

9 novembre 2022 15:19 — Par Patrice Lassère

[rms],1991/92.

Soit \(f\in\mathscr C^\infty(\mathbb R,\mathbb R)\) une fonction identiquement nulle sur \(\mathbb R\setminus ]-a,a[\). Montrer qu’il existe \(n\in\mathbb N\) tels que \(f^{(n)}\) admette au moins \(n+1\) zéros sur \(]-a,a[\).



[ID: 2707] [Date de publication: 9 novembre 2022 15:19] [Catégorie(s): Dérivabilité ] [ Nombre commentaires: 0] [nombre d'éditeurs: 1 ] [Editeur(s): Emmanuel Vieillard-Baron ] [nombre d'auteurs: 1 ] [Auteur(s): Patrice Lassère ]
Accordéon
Titre
Solution
Texte

Zéros des dérivées d’une fonctions \(\mathscr C^\infty\) à support compact
Par Patrice Lassère le 9 novembre 2022 15:19

Sans perdre de généralité on peut supposer \(a=1\) et \(f\) strictement positive sur \(]-1,1[\) (sinon \(n=0\) marche). Désignons par \(Z(f^{(n)})\) le nombre (peut-être infini) de zéros de \(f^{(n)}\) dans \(]-1,1[\). La régularité de \(f\) assure que \[\forall\,n\in\mathbb N\quad :\quad f^{(n)}(-1)=f^{(n)}(1)=0\] de telle sorte qu’en itérant le théorème de Rolle aux dérivées successives de \(f\) on a \[Z(f^{k+1})\geq Z(f^{(k)})+1\] soit après une recurrence immédiate \[Z(f^{(n+1)})\geq Z(f^{(n)})+1,\quad\forall\,n\in\mathbb N.\] Ainsi, \(Z(f^{(n)})\geq n,\forall\,n\in\mathbb N\) et la difficulté est de trouver un zéro supplémentaire.

La clé de cet exercice non trivial est le lemme suivant :

Preuve du lemme : Par une récurrence élémentaire, il est suffisant de prouver que pour tout \(n\in\mathbb N\) et tout \(\alpha\) réel hors de \(]-1,1[\) on a en posant \(g(x)=(x-\alpha)f(x)\) : \[Z(g^{(n+1)})\geq Z(f^{(n)})+1.\] Or \[\begin{aligned} g^{(n+1)}(x)&= (x-\alpha)f^{(n+1)}(x)+(n+1)f^{(n)}(x)\\ &= (x-\alpha)^{-n}\dfrac{d}{dx}\left( (x-\alpha)^{n+1}f^{(n)}(x)\right) \end{aligned}\] qui s’annule au moins une fois de plus sur \(]-1,1[\).\(\square\)

Nous sommes maintenant en mesure de résoudre l’exercice proposé. Soit \(p\) un entier tel que \[\left( \dfrac{3}{4}\right)^p<\rm{min}\left( \dfrac{f(0)}{f(1/2)},\dfrac{f(0)}{f(-1/2)}\right)\] et posons \(g(x)=(1-x^2)^{-p}f(x)\) avec \(g(-1)=g(1)=0\) ; par le lemme de division \(g\) est \(\mathscr C^\infty\) sur \(\mathbb R\) et par construction \[g(-1)<g(-1/2),\quad g(-1/2)>g(0),\quad g(0)<g(1/2)\quad \text{et}\quad g(1/2)>g(1).\] Il en résulte que \(g'\) s’annule au moins trois fois sur \(]-1,1[\) ; le lemme permet alors d’affirmer que \[Z(f^{(n+2p)})\geq Z(g^{(n)})+2p\] ce qui nous donne si \(n=1\) \[Z(f^{(2p+1)})\geq 2p+3.\] Ainsi \(f^{(m)}\) pour \(m\geq 2p+1\) possède au moins \(m+2\) zéros sur \(]-1,1[\).

Remarques : Cette solution nous laisse sur notre fin, en effet on ne comprends pas plus après sa lecture pourquoi ce mystérieux zéro supplémentaire va apparaitre et ne met pas vraiment non plus en évidence le role de la compacité du support de \(f\) qui est essentielle (la fonction \(f(x)=e^{-x^2}\) vérifie pour tout \(n\in\mathbb N\) : \(\lim_{x\to\pm\infty}f^{(n)}(x)=0\) mais on a \(Z(f^{(n)})=n\) ). Il serait vraiment trés intéressant d’avoir une autre preuve expliquant l’apparition de ce zéro supplémentaire.

En observant un peu plus en détail cette solution, il n’est alors pas difficile de montrer que pour tout entier \(k\geq 1\) les applications \(f^{(n)}\) admettront au moins \(n+k\) zéros dans \(]-a,a[\).

Le lemme de division est un outil essentiel pour beaucoup d’exercices d’analyse. Il dit en substance que (reprendre l’enoncé du Zuily et pourquoi la solution...........)


Sur l’inégalité de Kolmogorov \(M_1\leq 2\sqrt{M_0M_1}\). *

9 novembre 2022 15:19 — Par Patrice Lassère

Soit \(I\subset\mathbb R\) un intervalle et \(f\ :\ I\to\mathbb R\) une application deux fois dérivable telle que \[M_k:=\sup_{x\in I}\vert f^{(k)}(x)\vert<+\infty,\quad\forall\,k=0,1,2.\]

On suppose que \(I=\mathbb R\). Montrer que \(M_1\leq 2\sqrt{M_0M_1}\).

On suppose que \(I=[-c,c],\ c\in\mathbb R_+^\star\). Montrer que \[\vert f'(x)\vert\leq \dfrac{M_0}{c}+(x^2+c^2)\dfrac{M_2}{2c},\quad \forall\,x\in[-c,c].{\text{(\ding{56})}}\] \[M_1\leq 2\sqrt{M_0M_1}\quad\text{si}\quad c\geq \sqrt{\dfrac{M_0}{M_2}}.{\text{(\ding{52})}}\]

On suppose que \(I=(c,+\infty),\ c\in\mathbb R\). Montrer que \(M_1\leq 2\sqrt{M_0M_1}\).



[ID: 2709] [Date de publication: 9 novembre 2022 15:19] [Catégorie(s): Dérivabilité ] [ Nombre commentaires: 0] [nombre d'éditeurs: 1 ] [Editeur(s): Emmanuel Vieillard-Baron ] [nombre d'auteurs: 1 ] [Auteur(s): Patrice Lassère ]
Accordéon
Titre
Solution
Texte

Sur l’inégalité de Kolmogorov \(M_1\leq 2\sqrt{M_0M_1}\).
Par Patrice Lassère le 9 novembre 2022 15:19

Accordéon
Titre
Solution
Texte

Une série et une fonction
Par Patrice Lassère le 9 novembre 2022 15:19

On commence par construire une suite \((c_n)_n\subset\mathbb R_+^\star\) vérifiant \[\lim_{n\to\infty}c_n=+\infty\quad\text{et}\quad \sum_{n\geq 1}c_nb_n<\dfrac{1}{2}.\] Posons \(B:=\sum_{n\geq 1}b_n\) et désignons, pour tout \(k\in\mathbb N\), par \(N_k\) le plus petit entier vérifiant \[\sum_{n\geq N_k}b_n\leq \dfrac{B}{4^k}.\] La suite \((N_k)_k\) est bien définie. On pose alors \[c_n=\frac{2^k}{5B}\quad\text{pour tout}\quad N_k\leq n\leq N_{k+1},\] il est clair que \(c_n\to+\infty\) et \[\sum_{n\geq 1}c_nb_n=\sum_{k=0}^\infty\sum_{n=N_k}^{N_{k+1}-1}c_nb_n\leq \sum_{k=0}^\infty \dfrac{2^k}{5B}\sum_{n\geq N_k}b_n\leq \sum_{k=0}^\infty \dfrac{2^k}{5B}\dfrac{B}{4^k}=\dfrac{2}{5}.\] On considère alors les intervalles \(I_n=]a_n-c_nb_n,a_n+c_nb_n[\), la somme de leur longueurs est \(2\sum_n c_nb_n<1\) : il existe donc un réel \(x_0\in]0,1[\) qui n’est inclu dans aucun des intervalles \(I_n\). Nous allons montrer que \(f\) est dérivable en \(x_0\). \(x_0\in]0,1[\setminus\left( \cup_n I_n\right)\) implique que \(x_0\neq a_n\) pour tout \(n\in\mathbb N\), par conséquent \(f(x_0)=0\).

Soit \(x\in]0,1[\), s’il existe \(\in\mathbb N\) tel que \(x=a_n\) alors \[\left\vert\dfrac{f(x)-f(x_0)}{x-x_0}\right\vert=\dfrac{f(a_n)}{\vert a_n-x_0\vert}\leq \dfrac{b_n}{c_nb_n}=\dfrac{1}{c_n}\] sinon \[\left\vert\dfrac{f(x)-f(x_0)}{x-x_0}\right\vert=0.\] Puisque \(c_n\to\infty\) ces deux inégalités assurent que pour tout \(\varepsilon>0\) il n’existe qu’un nombre fini de réels \(x\in]0,1[\setminus\{x_0\}\) tels que \[\left\vert\dfrac{f(x)-f(x_0)}{x-x_0}\right\vert\geq \varepsilon.\] \(f\) est donc dérivable en \(x_0\) et \(f'(x_0)=0\).

Comme \(f\) est nulle sauf peut-être sur un ensemble dénombrable, si \(f\) est dérivable en un point \(x_0\) on aura nécessairement \(f(x_0)=f'(x_0)=0\). Si \((b_n)_n\) ne tends pas vers zéro, il n’est pas difficile de construire une autre suite \((\beta_n)_n\) telle que \(0 < \beta_n \leq b_n,\ \lim_n\beta_n=0\) et \(\sum_n\beta_n=\infty\). On construit alors la suite \((a_n)_n\) de telle sorte que les intervalles \(I_n:=]a_n-\beta_n,a_n+\beta_n[\) rencontrent chaque point de \(]0,1[\) une infinité de fois (ceci est possible puisque la somme des longueurs de intervalles est \(\sum_n2\beta_n=\infty\)). Avec ce choix, pour tout \(x_0\in]0,1[\) vérifiant \(f(x_0)=0\) (si \(f(x_0)\neq 0\) nous avons déja remarqué que \(f\) ne peut être dérivable en ce point) et tout \(\varepsilon>0\) il existe \(n\in\mathbb N\) tel que \(\beta_n<\varepsilon\) et \(x_0\in I_n\) qui implique \[\left\vert\dfrac{f(xa_n)-f(x_0)}{a_n-x_0}\right\vert\geq\dfrac{b_n}{\beta_n}\geq 1.\] \(0\) étant la seule valeur possible pour \(f'(x_0)\), \(f\) n’est pas dérivable en \(x_0\).


Accordéon
Titre
Solution
Texte

Un fameux théorème d’Émile Borel
Par Patrice Lassère le 9 novembre 2022 15:19

Soient \(n\in\mathbb N^\star, k\in\{0,\dots,n-1\}\). Avec la formule de Leibnitz \[f_n^{(n)}(x)=\dfrac{a_n}{n!}\sum_{p=0}^k C_k^pn(n-1)\dots(n-p+1)x^{n-p}\lambda_n^{k-p}\varphi^{(k-p)}(\lambda_n x)\] De plus \(\varphi^{(k-p)}(\lambda_n x)=0\) pour \(\vert x\vert\geq 2/\vert\lambda_n\vert\) d’où \[\begin{aligned} \sup_{x\in\mathbb R}\left\vert f_n^{(k)}(x)\right\vert&\leq \dfrac{\vert a_n\vert}{n!}\sum_{p=0}^k C_k^p \dfrac{n!}{(n-p)!}\vert x\vert^{n-p}\vert\lambda_n\vert^{k-p}\sup_{x\in\mathbb R}\vert\varphi^{(k-p)}(\lambda_n x)\vert\\ &\leq {n!}\sum_{p=0}^k \dfrac{C_k^p}{(n-p)!}\left( \dfrac{2}{\vert\lambda_n\vert}\right) ^{n-p}\vert\lambda_n\vert^{k-p}\sup_{\lambda_n x\in[-2,2] }\vert\varphi^{(k-p)}(\lambda_n x)\vert&\\ &\leq \dfrac{\vert a_n\vert M_n}{\vert\lambda_n\vert^{n-k}}\sum_{p=0}^k \dfrac{C_k^p 2^{n-p}}{(n-p)!}&(\text{\ding{56}}) \end{aligned}\]\(\displaystyle M_n:=\max_{0\leq k\leq n-1}\left( \sup_{\lambda_n x\in[-2,2] }\vert\varphi^{(k-p)}(\lambda_n x)\vert\right)\). D’autre part, avec les majorations grossières pour \(0\leq p\leq k\leq n-1\) : \[C_k^p\leq k!\leq (n-1)!,\quad \dfrac{2^{n-p}}{(n-p)!}\leq 2^n\] supposons en outre \(\vert\lambda_n\vert\geq 1,\ \forall\,n\in\mathbb N\), alors \(1/\vert\lambda_n\vert^{n-k}\leq 1/\vert\lambda_n\vert\), et finalement () devient \[\sup_{x\in\mathbb R}\left\vert f_n^{(k)}(x)\right\vert\leq \dfrac{\vert a_n\vert M_n}{\vert\lambda_n\vert^{n-k}}\sum_{p=0}^k k!2^n\leq \dfrac{\vert a_n\vert M_n}{\vert\lambda_n\vert}2^nn!.\] de cette dernière inégalité, nous avons \[\vert\lambda_n\vert\geq\max\left( 1,\,M_n\vert a_n\vert 4^n n!\right){(\text{\ding{52}}_n)}\] qui implique \[\sup_{x\in\mathbb R}\left\vert f_n^{(k)}(x)\right\vert\leq 2^{-n},\quad\forall\,0\leq k\leq n-1.{(\text{\ding{54}})}\] En résumé, toute suite \((\lambda_n)_n\) de réels vérifiant (\(_n\)) convient.

Sous ces choix, considèrons l’application \(f:=\sum_{n\geq 0}f_n\). L’inégalité () (avec \(k=0\)) assure la normale convergence sur \(\mathbb R\) de la série de fonctions définissant \(f\) qui est donction définie et continue sur \(\mathbb R\).

Maintenant, pour \(k\geq 1\), en écrivant \[\sum_k f_n^{(k)}=\sum_{n\geq k}f_n^{(k)}+\sum_{n>k}f_n^{(k)}\] l’inégalité \(\sup_\mathbb R\vert f_n^{(k)}\vert<2^{-n}\) pour \(n>k\) implique que la série \(\sum_{n}f_n^{(k)}\) est normalement convergente sur \(\mathbb R\) ; ainsi (c’est le théorème de Weierstrass sur la dérivation des séries de fonctions), \(f\) est indéfiniment dérivable sur \(\mathbb R\) et on peut dériver sous le \(\sum\) : \[\begin{aligned} f^{k}(x)&=\sum_{n=0}^kf_n^{(k)}(x)+\sum_{n>k}f_n^{(k)}(x)\\ &=\left( \sum_{n=0}^k +\sum_{n>k} \right) \left( \sum_{p=0}^k \dfrac{a_n}{n!}C_k^p\left( x^n\right)^{(k)}\left( x^n\varphi(\lambda_n x)\right)^{(n-p)} \right),\quad x\in\mathbb R,\ n\in\mathbb N. \end{aligned}\] Maintenant, comme \(\varphi\) est constante égale à \(1\) sur \([-1,1]\), ses dérivées d’ordre supérieur à \(1\) sont identiquement nulles sur \(]-1,1[\) donc en particulier à l’origine. De même \[\left( x^n \right)^{(k)}\big\vert_{x=0}=\begin{cases} n!\quad&\text{si }\ n=k\\ 0\quad&\text{sinon.}\end{cases}\] De là, pour évaluer \(f^{(k)}(0)\), seul un terme n’est pas nul dans l’expression précédente, il reste précisément : \[f^{(k)}(0)=\dfrac{a_k}{k!} \left( x^n \right)^{(k)}\varphi(\lambda_n x)\big\vert_{x=0}=a_k,\] et \(f\) possède bien les propriétés désirées.


\(\sum_{k=1}^n\,(-1)^{k+1}C_n^kk^n=(-1)^{n+1}n!\) *

9 novembre 2022 15:19 — Par Patrice Lassère

Montrer que pour tout \(n\in\mathbb N\) \[\sum_{k=1}^n\,(-1)^{k+1}C_n^kk^n=(-1)^{n+1}n!\] On peut remarquer que \((e^{kx})^{(n)}_{x=0}=k^n\)...



[ID: 2715] [Date de publication: 9 novembre 2022 15:19] [Catégorie(s): Dérivabilité ] [ Nombre commentaires: 0] [nombre d'éditeurs: 1 ] [Editeur(s): Emmanuel Vieillard-Baron ] [nombre d'auteurs: 1 ] [Auteur(s): Patrice Lassère ]
Accordéon
Titre
Solution
Texte

\(\sum_{k=1}^n\,(-1)^{k+1}C_n^kk^n=(-1)^{n+1}n!\)
Par Patrice Lassère le 9 novembre 2022 15:19

Nous avons \[\begin{aligned} \sum_{k=1}^n\,(-1)^{k+1}C_n^kk^n&=-\lim_{x\to 0}\dfrac{d^n}{dx^n}\left( \sum_{k=0}^n(-1)^kC_n^k e^{kx}\right) \\ &=-\lim_{x\to 0}\dfrac{d^n}{dx^n}\left( (1-e^x)^n\right) \\ &=-\lim_{x\to 0}\dfrac{d^n}{dx^n}\left( -x-\dfrac{x^2}{2!}-\dfrac{x^3}{3!}-\dots\right)^n \\ &=(-1)^{n+1}\lim_{x\to 0}\dfrac{d^n}{dx^n}\left( x^n+n\dfrac{x^{n+1}}{2}+\dots\right) \\ &=(-1)^{n+1}n! \end{aligned}\]


;
Success message!